HELP ASAP PLS ILL MARK YOU BRAINLIEST

HELP ASAP PLS ILL MARK YOU BRAINLIEST

Answers

Answer 1
The answer should be C

The origin has moved two to the right and two up. This means there must be (x-2) and a +2 at the end of the equation. The function visibly is a cube root so that all matches function C
Answer 2

Answer:

c is the answer.


Related Questions

the inverse of f(x)=2x-10

Answers

The inverse of the function f(x) = 2x - 10 is f-1(x) would be x/2 + 5.

Lets try to solve it,

We will be using the inverse function to solve this.

f(x) =[tex]2x - 10[/tex]

Replace f(x) with y.

[tex]y = 2x - 10[/tex]

Interchange x and y

[tex]x = 2y - 10[/tex]

Solve for y

[tex]y = (x + 10) / 2[/tex]

Replace y with f-1(x).

[tex]f-1(x) = x/2 + 5[/tex]

Thus, the inverse of the function f(x) = 2x - 10 is f-1(x) = x/2 + 5

Learn more about inverse function on:

https://brainly.com/question/11735394

#SPJ10

Please help!!!!!!!!!!!!!!!!!!!!!

Answers

The values for y are i, 0, √3 and 2√2

What is function?

An expression, rule, or law that defines a relationship between one variable (the independent variable) and another variable.

Given:

F(x)= y = √(x-5) -1

At x=5,

y=  √(5-5) -1

y= i

At x= 6

y= √(6-5) -1

y=0

At x= 9

y= √(9-5) -1

y=√3

At x= 14

y= √(14-5) -1

y=√8= 2√2

Learn more about function here:

https://brainly.com/question/12431044

#SPJ1

I'm having problems because our teacher is making us do this monstersety 391.92x10372.45-918+12.5

Answers

Answer:

4064265.104

Step-by-step explanation:

Use PEMDAS :

Parentheses, Exponents, Multiplication, Division, Addition, Subtraction.

We only have multiplication and addition/subtraction to do. Multiplication and Division can go in either order depending on which comes first reading left to right. It's the same with Addition and Subtraction: we will first subtract 918 and then add 12.5.

Let's start by Multiplying. 391.92 x 10372.45 = 4065170.604.

Now, subract 918. 4065170.604-918 = 4064252.604.

Finally, add back 12.5. 4064252.604. +12.5 = 4064265.104 !

6root27+root243 simplify

Answers

Answer:

6√27 + √243

6√9 x 3 + √81 x 3  

6 x 3 √3 + 9√3

18√3 + 9√3

27√3

what is the factor each expression of x2 - 2x

Answers

Answer:

x(x - 2)

Step-by-step explanation:

x² - 2x ← factor out x from each term

= x(x - 2)

Which figures are polygons?
x
Figure A
Select each correct answer.
Figure B
O Figure A
O Figure B
O
Figure C
O
Figure D
Figure C
Figure D

Answers

Answer:

The answer is figure A, B, C, D.

Step-by-step explanation:

Because, polygon means a shape with more then 1 side, and in your pic all of the figures have more than one side, so that's the answer.

Type the correct answer in each box. Use numerals instead of words. If necessary, use / for the fraction bar. Consider the given table. x -2 2 5 9 f(x) 5 17 26 ?

The "?" in the table represents the number_because the average rate of change on every interval of the function is_ .

Answers

A line is a one-dimensional shape that is straight, has no thickness, and extends in both directions indefinitely. The value of f(x) when the value of x is 9 is 38.

What is the equation of a line?

A line is a one-dimensional shape that is straight, has no thickness, and extends in both directions indefinitely. The equation of a line is given by,

y =mx + c

where,

x is the coordinate of the x-axis,

y is the coordinate of the y-axis,

m is the slope of the line, and

c is the y-intercept.

The given data points are in a linear relation, therefore, if we find the slope of the line on which these data points will be located. Then the slope will be,

m = (17-5)/(2+2) = 3

Now, equate any point and the slope in the general equation of line, therefore, the equation will be,

y = 3x + c

17 = 3(2) + c

17 - 6 = c

c = 11

Further, the value of f(x) when the value of x is 9 will be,

f(x) = 3x + 11

f(9) = 3(9) + 11

f(9) = 38

Hence, the value of f(x) when the value of x is 9 is 38.

Learn more about Equation of Line:

https://brainly.com/question/21511618

#SPJ1

What number is the arrow pointing at?

Answers

Answer:

170

Step-by-step explanation:

each little mark is adding 20 to the 100 you see there and because the arrow is pointing somewhere in between two of the marks, it means it's just adding 10. Now you have to add 100 + the marks + 10

* remember you have three marks and each one of them is equal to 20 so when you add them, you get 60.

100 + 60 + 10 = 170

What is the simple interest that is missing from the table? Use the formula mc012-1.jp




$375
r
2.2%
t
3 years
$25.00
$24.75
$2,475.75
$2,500.00

Answers

The simple interest is $24.75.

What is the simple interest?

The simple interest that is paid on an amount of money is a function of the amount deposited, time and interest rate.

Simple interest = amount deposited x time x interest rate

$375 x 0.022 x 3 = $24.75

To learn more about simple interest, please check: https://brainly.com/question/27328409

#SPJ1

Simplify this expression.
[tex]\\\\(\sqrt{2} + \sqrt{3})(\sqrt{5} - \sqrt{7} \\A: \sqrt{5} + \sqrt{-2} \\B: \sqrt{10} + \sqrt{15} - \sqrt{14} - \sqrt{21} \\C: 2\sqrt{5} - 2\sqrt{7} \\D: 2\sqrt{5} + 3\sqrt{5} - 2\sqrt{7} - 3\sqrt{7}[/tex]

Answers

By applying algebra, radical and power properties, we find that the simplified form of (√2 + √3) · (√5 - √7) is equal to √10 + √15 - √14 - √21. (Correct choice: B)

How to simplify a product of two binomials with radical numbers

Herein we need to apply algebra and radical and power properties to simplify the expression described in the statement, whose procedure is shown below:

(√2 + √3) · (√5 - √7)     Given(√2 + √3) · √5 + (√2 + √3) · (- √7)     Distributive property√2 · √5 + √3 · √5 + √2 · (- √7) + √3 · (- √7)     Distributive property√10 + √15 - √14 - √21     (- a) · b = - a · b/ √a · √b = √a · b/Result

By applying algebra, radical and power properties, we find that the simplified form of (√2 + √3) · (√5 - √7) is equal to √10 + √15 - √14 - √21. (Correct choice: B)

To learn more on radical numbers: https://brainly.com/question/13184531

#SPJ1

Answer:

B is correct

Step-by-step explanation:

Cus I said so

If j is 53 degrees and l is 29 how much is k?

Answers

Answer:

98

Step-by-step explanation:

There are 180 degrees in a triangle.

180 - 53 - 29 = 98

Gcf of 24x^5 - 56x^3 + 16x

Answers

Answer:

8x

Step-by-step explanation:

Greatest common factor is the factor of 24, 56 and 16. Since 16x only has the power of 1, then then the answer should be on power of 1.

Answer:

Given the 24x⁵ - 56x³ + 16x contains numbers and variables, there are two steps to finding the GCF. Find the GCF for the numeric part, then find the GCF for the variable part.

Steps to find the LCD of 24x⁵ - 56x³ + 16x

Find the LCD for the numerical part 24, - 56, 16x Find the LCD of the variable part x⁵, x³, x¹.Multiply the values with each other

Find the common factors of the numerical part: 24, −56, 16

The factors for 24 are −24,−12,−8,−6,−4,−3,−2,−1,1,2,3,4,6,8,12,24.

The factors for −56  are−56,−28,−14,−8,−7,−4,−2,−1,1,1,2,2,4,4,7,7,8,8,14,14,28,28,56,56.

The factors for 16  are −16,−8,−4,−2,−1,1,2,4,8,16.

List all the factors 24,−56,16 to find the common factors.

24:−24,−12,−8,−6,−4,−3,−2,−1,1,2,3,4,6,8,12,24−56:−56,−28,−14,−8,-7,−4,−2,−1,1,1,2,2,4,4,7,7,8,8,14,14,28,28,56,5616:−16,−8,−4,−2,−1,1,1,4,8,16

The common factors for 24,−56,16 are −8,−4,−2,−1.−8,−4,−2,−1

The GCF of the numerical part is 8.

[tex]\bf{GCF_{Numerical}=8 }[/tex]

Then find the common factors for the variable part:

x⁵, x³, x

The factors for x⁵ are:

x · x · x · x · x

The factors for are:

x · x · x

The factors for are:

x

List all the factors x⁵, x³, x¹ to find the common factors.

x⁵ = x · x · x · x · x

x³ = x · x · x

x¹ = x

The common factor of the variables x⁵, x³, x¹ is x.

The LCD of the variable part is x.

[tex]\bf{GCF_{Numerical}=x }[/tex]

Multiply the LCD of the numerical part 8 and the GCF of the variable part x.

8x

find the range for the given domain: {-3,-1 and 3} and the function is y=3x^2-2x+1.

Answers

Answer:

D

Step-by-step explanation:

The domain are the x values. When you plug those values into the equation, you get 34, 6, 22.

3(-3²) - 2(-3) + 1                     Negative * Negative = Positive

3(9) + 6 + 1

34

3(-1²) - 2(-1) + 1

3 + 2 +1

6

3(3²) -2(3) + 1

3(9) - 6 + 1

22

smone answer ASAP please.

Answers

The measure of the angle x is 117 degrees

Line geometry.

To determine the value of x in the given diagram, we will use the alternate angles theorem

The value of x is expressed as shown;

p + q= x

If the value of p = 51, then;

51 + q. = x

Determine the value of. q

q = 108 - 42

q = 66

Determine the value of x

x = 66 + 51

x = 117 degrees

Hence the measure of the angle x is 117 degrees

Learn more on angles here: https://brainly.com/question/25716982

#SPJ1

J
Question 1 of 10
Each equation given below describes a parabola. Which statement best
compares their graphs?
x = 5y²
x=3y²
OA. Both parabolas open upward, and x = 3y2 is wider than
x = 5y².
B. Both parabolas open to the right, and x = 3y² is wider than x = 5y².
OC. Both parabolas open upward, and x = 5y2 is wider than
x = 3².
OD. Both parabolas open to the right, and x = 5y² is wider than x = 3y².
SUBMIT

Answers

Answer:

D

Step-by-step explanation:

x = 5y²

x = 3y²

Here x is a function of y and so the parabola is open either to right or left.

The coefficient of y² is positive. So the parabola is open to right.

Both parabolas are open to right and  x = 5y² is wider than x = 3y².

The figure below shows a square ABCD and an equilateral triangle DPC:

ABCD is a square. P is a point inside the square. Straight lines join points A and P, B and P, D and P, and C and P. Triangle D

Ted makes the chart shown below to prove that triangle APD is congruent to triangle BPC:

Statements Justifications
In triangles APD and BPC; DP = PC Sides of equilateral triangle DPC are equal
In triangles APD and BPC; AD = BC Sides of square ABCD are equal
In triangles APD and BPC; angle ADP = angle BCP Angle ADC = angle BCD = 90° so angle ADP = angle BCP = 60°
Triangles APD and BPC are congruent SAS postulate
What is the error in Ted's proof? (1 point)

He writes the measure of angles ADP and BCP as 60° instead of 45°.
He uses the SAS postulate instead of AAS postulate to prove the triangles congruent.
He writes the measure of angles ADP and BCP as 60° instead of 30°.
He uses the SAS postulate instead of SSS postulate to prove the triangles congruent.

Answers

Answer:

Which of the following completes Ted's proof?

In square ABCD; angle ADC = angle BCD

In square ABCD; angle ADP = angle BCP

In triangles APD and BPC; angle ADC = angle BCD

In triangles APD and BPC; angle ADP = angle BCP\

which expression is equivalent to 354x + 332, If x=0?

Answers

Answer:

10x + 332

when, x=0

Step-by-step explanation:

Please give me brainlist.

The sum of two numbers is 48. If one number is three more than twice the second number, which of the following is the
smaller number?

O 24
O 15
O 30
O 33

Answers

Answer:

15

Step-by-step explanation:

"one number is three more than twice the second number"

15 * 2 = 30

30 + 3 = 33

15 + 33 = 48

Answer:

15

Step-by-step explanation:

The key to most problems of this type is carefully translating the word problem into math ... an equation.

"The sum of two numbers is 48."

"Sum": Add

"two numbers": we'll need two numbers... let's call them x & y

"is": =

"48": 48

So, the first sentence means:  [tex]x+y=48[/tex]

Note that we have two unknowns here.  To solve for them, we'll need another equation... at least as many equations as unknowns (2 unknowns, needs 2 equations).

"If one number is three more than twice the second number..."

"one number": x

"is": =

"three more than": +3

"twice": 2*

"the second number": y

So, the second sentence introductory clause means: [tex]x=2y+3[/tex]

Systems of equations

We have a system of equations. We need as many equations as we have unknowns.  We currently have two equations, and two unknowns.  Let's try to solve the system:

[tex]x+y=48[/tex]

[tex]x=2y+3[/tex]

There are two main methods to solve systems of equations:  Substitution & Elimination

To use the Substitution Method, we'll need to isolate one of the variables in one of the equations, substitute, and solve.

To use the Elimination Method, we'll need to align like terms, find or build matching coefficients with opposite signs, add to eliminate, and solve.

Given that equation 2 already has x isolated, this system is already ready to use the Substitution method, so let's do that.

Solving a system with the Substitution Method

[tex]x+y=48\\x=2y+3[/tex]

[tex](2y+3) + y = 48[/tex]

[tex]2y+3 + y = 48[/tex]

[tex]3y+3 = 48[/tex]

[tex]3y= 45[/tex]

[tex]y= 15[/tex]

To find x, substitute back into one of the equations:  Equation 2 already has x isolated, so it will be easy to solve for x.

[tex]x=2(15)+3[/tex]

[tex]x=30+3[/tex]

[tex]x=33[/tex]

So the solution we found is x=33 and y=15.

Check to make sure it satisfies the original conditions:

The sum of the numbers is 48:

[tex]x+y=(33)+(15)\\x+y=48[/tex]

One number is three more than twice the second number

[tex]x=2y+3[/tex]

[tex](33)\text{ ?=? }2(15)+3[/tex]

[tex]33=33[/tex]

Our answer satisfies all of the original conditions, so our answer works.

Answering the question

The question finally asks, "...which of the following is the smaller number?"

Of the two numbers, the smaller number is 15.

1. Find the range for the given domain: {-2, -1, and 0} and the function is y = 3x^2
2. Find the range of the given function y = 8x + 1, where x > 2
3. Find the range of the given function y = 5x + 2, where x > -2


can someone explain how to do these?

Answers

1. The range of the given function y = 3x² for the given domain: {- 2, - 1, and 0} is {12, 3, 0}.

2. The range of the given function y = 8x + 1, where x > 2 is (17, ∞).

3. The range of the given function y = 5x + 2, where x > -2 is (- 8, ∞).

We know that the range of a function is the entire set of all possible values for the dependent variable (typically y), after substituting the domain.

1. For the function y = 3x²,

Here the domain is {- 2, - 1, and 0}. So, we have to find the y values for the x values - 2, - 1, and 0 to find the range of the function.

Now, at x = - 2,

y = 3(- 2)² = 3 × 4 = 12

At x = - 1,

y = 3(- 1)² = 3 × 1 = 3

At x = 0,

y = 3(0)² = 3 × 0 = 0

So, the range of this function is {12, 3, 0}.

2. Now, for the function y = 8x + 1,

At x = 2,

y = 8 × 2 + 1 = 16 + 1 = 17

Since the domain is x > 2, we can say that the range set will contain values greater than 17.

So, the range of this function is (17, ∞).

3. For the function y = 5x + 2,

Now, at x = - 2,

y = 5 × (- 2) + 2 = - 10 + 2 = - 8

Since the domain is x > - 2, we can say that the range set will contain values greater than - 8.

So, the range of this function is (-8, ∞).

1. Therefore, the range of the given function y = 3x² for the given domain: {- 2, - 1, and 0} is {12, 3, 0}.

2. The range of the given function y = 8x + 1, where x > 2 is (17, ∞).

3. The range of the given function y = 5x + 2, where x > -2 is (- 8, ∞).

Learn more about the range of a function here -

https://brainly.com/question/27954428

#SPJ10

L
N
(x-4) in. O
(x-3) in.
(x + 2) in.
x in.
K
Which value of x would make NO || KJ?
1
6
08
O 10

Answers

Answer:

x = 8

Step-by-step explanation:

[tex]\sf If\:\: \overline{NO} \parallel \overline{KJ}\:\:then\:\: \triangle LNO \sim\triangle LKJ[/tex]

Therefore:

[tex]\implies \sf \overline{LN} : \overline{LO} = \overline{LK} : \overline{LJ}[/tex]

[tex]\implies (x-3):(x-4)=(x-3)+(x+2):(x-4)+x[/tex]

[tex]\implies \dfrac{x-3}{x-4}=\dfrac{2x-1}{2x-4}[/tex]

[tex]\implies (x-3)(2x-4)=(2x-1)(x-4)[/tex]

[tex]\implies 2x^2-10x+12=2x^2-9x+4[/tex]

[tex]\implies -10x+12=-9x+4[/tex]

[tex]\implies 12=x+4[/tex]

[tex]\implies x=8[/tex]

the area of square ABCD is 10cm2 show that x^2+6x=a where a is a constant to be found

Answers

The value of a is 1 if the area of square ABCD is 10 cm² showing that x²+6x=a where a is a constant.

What is the area of the rectangle?

It is defined as the area occupied by the rectangle in two-dimensional planner geometry.

The area of a rectangle can be calculated using the following formula:

Rectangle area = length x width

The question is incomplete.

The question is:

The area of square ABCD is 10 cm² showing that x²+6x=a where a is a constant to be found.

The missing figure is attached; please refer to the picture.

As we can see in the figure we have given a rectangle.

Area of rectangle = (3 + x)(3 + x)

10 = (3 + x)(3 + x)

or

10 = (3 + x)²

10 = 9 + 6x + x²

After rearranging:

x² + 6x = 1

We have equation:

x² + 6x = a

After comparing:

a = 1

Thus, the value of a is 1 if the area of square ABCD is 10 cm² showing that x²+6x=a where a is a constant.

Learn more about the rectangle here:

https://brainly.com/question/15019502

 

#SPJ1

write an equation in slope intercept form of the line with the given characteristics: a) passes through (-3,0) and (-5,-3)

Answers

The equation of the line in slope-intercept form is [tex]\bf{y= \frac{3}{2} x + \frac{9}{4}} [/tex].

What is slope-intercept form?y=mx +c, where m is the slope and c is the y-intercept

StepsFind the slopeSubstitute value of the slope into the equationSubstitute a pair of coordinatesSolve for cRewrite equation with value of m and c

Working

The first step is to calculate the slope of the line.

[tex]\boxed{ slope = \frac{y _{1} - y_2 }{x_1 - x_2} }[/tex]

Slope

[tex] = \frac{ - 3 - 0}{ - 5 -( - 3)} [/tex]

[tex] = \frac{ - 3}{ - 5 + 3} [/tex]

[tex] = \frac{ - 3}{ - 2} [/tex]

[tex] = \frac{3}{2} [/tex]

Substitute the value of m into y= mx +c:

[tex]y = \frac{3}{2} x + c[/tex]

Next, substitute a pair of coordinates that the line passes through. Here, I will substitute (-3, 0) into the equation.

When x= -3, y= 0,

[tex]0 = \frac{3}{2} ( - 3) + c[/tex]

Find the value of c:

[tex]0 = - \frac{9}{2} + c[/tex]

[tex]c = \frac{9}{2} [/tex]

Substitute the value of c into the equation:

Thus, the equation of the line is [tex]y = \frac{3}{2} x + \frac{9}{2} [/tex].

To learn more about slope-intercept form, check out: https://brainly.com/question/24436844.

Find the highest common factor [HCF] of 10 and 17

Answers

Answer

Step by step explanation

1 is the answer for this question

What is this Simplify as √16y¹6

Answers

Answer:

[tex]6y^1^6[/tex]

Step-by-step explanation:

I hope this has Helped :)

Solve the equation.
4.5=0.5n

Answers

Divide by 0.5 on both sides
4.5 divided by 0.5 = 9
Have a nice day

HELP !!!
Which of the following best describes the slope of the line below?
• A. Negative
O B. Zero
• C. Positive
D. Undefined

Answers

The answer to this is negative

i need help what's 8*3^2 i will mark brainiest if someone can answer it in 72-minute s

Answers

Answer:

72

Step-by-step explanation:

8 x 3² = 8 x 3 x 3

          = 8 x 9

          = 72

[P.S. If you meant (8 x 3)² the answer is:

(8 x 3)² = (24)²

            = 24 x 24

            = 576     ]  

A
D
4
324
OX+0
+
54324
-2
-3
&
B
h
23
X
Which rule describes the x-coordinates in the translation?
A
27
image

Answers

Answer:

x + 6

Step-by-step explanation:

consider the x- coordinates of the right angle

x- coordinate of A = - 4

x- coordinate of B = 2

then

| 2 - (- 4) | = | 2 + 4 | = | 6 | = 6

the x- coordinate has been shifted 6 units to the right , so the rule is

x + 6

A table of data is given.
x f(x)
-2 71
-1 13
0 3
1 0.6
2 0.1
Which exponential model best represents the data?
Of(x) = 3(1.2)^x
Of(x) = 2(0.3)^x
Of(x) = 2(3)^x
Of(x)=3(0.2)^x

Answers

Answer:

The choice D.

[tex]f(x) = 3 {(0.2)}^{x} [/tex]

Answer:

D

Step-by-step explanation:

Order of elimination:

1. the graph is going down exponentially. This means that the base is less than 1.

A and C are eliminated.

2. When x is equal to 0, f(x) = 3.

Substitute x for 0, and you'll find out that the only one possible is D.

Topic: Speed
Subject: Mathematics
Answer the Question

Q1: Express each of the following in km/h

a: 8.4 km/min b: 315 m/s
c: 242 m/min d: 125 cm/s

Answers

Hope this helps you

........

Other Questions
Which character in American Born Chinese is an example of a foil?OA. The Monkey KingO B. Wei-ChenOC. Suzy NakamuraO D. Jin Wang CorrectCorrect! Gotchu ^ Explain two contrasting religious beliefs about the nature of marriage.In your answer you must refer to one or more religious traditions. GIVING BRAINLIEST NOW FOR CORRECT AWNSERReemplaza el complemento directo por el pronombre de objeto directo: "Los estudiantes entregan los libros.Los estudiantes _____ entregan".Your answer:loslalaslo Need help simple explanation if possible A 2-column table with 6 rows. The first column is labeled x with entries negative 3, negative 2, negative 1, 0, 1. The second column is labeled f of x with entries 15, negative 5, 0, 5, 0, negative 5. Which of these are core properties of kanban? (select all that apply) 1 point retrospectives daily stand-up limit work in progress visualize the work manage flow What is the answer????? I SEEK INFO.It's been a while since I've thought about joining the Army, BUT I know nothing about what I should do, or the process. Can I assist college while being in the army or.. how does it work?Anything help, Thank you! during which software development phase do users begin using an app? The graph of F(x), shown below, resembles the graph of G(x)= x, but it hasbeen changed somewhat. Which of the following could be the equation ofF(x)?A. F(x) = 3(x+4)2 +4B. F(x)=3(x-4)2 - 4C. F(x)=-3(x+4) +4D. F(x) = -3(x-4) +4 Which sentence is the most formal?OA. I think it would be really fun to work at your company.OB. I would like to apply for the administrative position at yourcompany.OC. Hi! Do you have any job openings at your work yet?OD. Please hiring my person for the open position you got in yourestablishment.SUBMIT Consider what the host means when he says,"Every statement I've made has nothing to do with what you have done--hasnothing to do with decisions you've made. Which graph represents the function h(x) = |x| + 0.5?On a coordinate plane, an absolute value graph has a vertex at (0, 1.5).On a coordinate plane, an absolute value graph has a vertex at (negative 0.5, 0).On a coordinate plane, an absolute value graph has a vertex at (0, 0.5).On a coordinate plane, an absolute value graph has a vertex at (negative 1.5, 0). glomerular capillaries and podocytes together form the: A researcher investigated the relationship between alcohol intake and reaction time using a driving simulation task. Participants drank either one or three ounces of pure alcohol with the taste masked by diluting the alcohol in a strong-tasting fruit punch. Their reaction time was measured in terms of the speed with which they braked on seeing a red light during the simulated driving task. In this case, the dependent variable in the experiment is the _____. The longitudes that are on the same line are _____ Ignore this it a temporary note for me: SPSstudents Who was the last ruler of Mughal Empire? What should Giovanna add to the paragraph that would make it a complete argument?At the beginning of the paragraph, she should add a quotation.At the beginning of the paragraph, she should add a topic sentence.At the end of the paragraph, she should add a relevant counterclaim.At the end of the paragraph, she should add a link back to the claim. Solve for x (12x-7)/4=(5x+18)/3